Đến nội dung

letrongvan nội dung

Có 207 mục bởi letrongvan (Tìm giới hạn từ 09-05-2020)



Sắp theo                Sắp xếp  

#373864 Tổng hợp các bài toán Tích phân

Đã gửi bởi letrongvan on 29-11-2012 - 22:14 trong Giải tích

Cảm ơn sự tham gia của dark templar. Mọi người cùng tham gia giải nào. Sau đây là một bài dành cho Đại học.

Bài 5: Tính tích phân: $$I_{5}=\int_{-1}^{1}\frac{dx}{\sqrt{1-x^{2}}}$$

hình như mình sai rồi, tích phân k tồn tại khi hàm duới dấu tích phân không bị chặn, mới học xong 2 hôm nên cũng lờ mờ chẳng biết đúng hay sai



#414706 Tổ hợp tuyến tính

Đã gửi bởi letrongvan on 24-04-2013 - 23:03 trong Đại số tuyến tính, Hình học giải tích

Cái này bạn nên xem lại giáo trình, các định lý ý, nên đọc hết, không hiểu hãy hỏi




#445936 Tính xs để cho viên đạn do xạ thủ 2 bắn trúng mục tiêu

Đã gửi bởi letrongvan on 28-08-2013 - 18:03 trong Xác suất - Thống kê

Xác suất để người thứ nhất và thứ hai cùng bắn trúng mục tiêu là $P_1=0,6.0,8/3=0,16$.

Xác suất để người thứ hai và người thứ ba cùng bắn trúng mục tiêu là $P_2=0,8.0,9/3=0,24$.

Xác suất để người thứ hai bắn trúng mục tiêu khi có hai viên trúng đích là $P=P_1+P_2=0,4$.

Không biết đúng k nhỉ???> :D

Sai rồi :D

mỗi người bắn có 1 phát thôi mà

bài toán trên có 2 trường hợp 1,2 và 2,3 thôi




#440938 Tính giới hạn $\lim_{x\to \infty }{\f...

Đã gửi bởi letrongvan on 07-08-2013 - 08:43 trong Giải tích

Dùng quy tắc L'hospital, đạo hàm cả tử và mẫu thì giới hạn bằng 0




#427012 Tìm miền hội tụ:$\sum\limits_{n=1}^\infty...

Đã gửi bởi letrongvan on 14-06-2013 - 02:11 trong Giải tích

 
Khi_ $x=\pm \sqrt{e}+3$ _, chuỗi đã cho có dạng_ $\sum_{n=1}^{+\infty}\left ( -1 \right )^n \cdot \dfrac{n!}{n^n} \cdot e^n$
Ta có__ $\dfrac{|u_{n+1}|}{|u_{n}|} = \dfrac{e^{n+1}.\left( n+1 \right )!}{\left ( n+1 \right )^{n+1}} \cdot \dfrac{n^n}{e^n.n!} = \dfrac{e .n^n}{\left (n+1 \right )^n} = \dfrac{e}{\left (1+\frac{1}{n} \right )^n} \rightarrow 1$
 Tuy nhiên vì__$\left (1+\dfrac{1}{n}  \right )^n < e$ __nên__$\dfrac{|u_{n+1}|}{|u_{n}|} > 1$
Vậy__ $\sum_{n=1}^{+\infty}\left ( -1 \right )^n \cdot \dfrac{n!}{n^n} \cdot e^n$__phân kỳ .
 
Kết luận , miền hội tụ của chuỗi đã cho là_ ($-\sqrt{e} +3$ ; $\sqrt{e}+3$) .

Tiêu chuẩn nào đây??? Áp dụng tiêu chuẩn Da Lăm Be cho chuỗi đan dấu, bái phục bạn rồi. Ngoài laibniz ra không được áp dụng những cái này, tiêu chuẩn tích phân, cô-si hay đa lăm be đầu là các tiêu chuẩn áp dụng cho chuỗi dương, sai bét rồi!




#427349 Tìm miền hội tụ:$\sum\limits_{n=1}^\infty...

Đã gửi bởi letrongvan on 15-06-2013 - 07:02 trong Giải tích

Bạn hãy xem lại kiến thức của mình và đọc lại những bài ở trên, xem người ta nói gì, viết gì, đừng chỉ đọc đoạn giữa rồi phán xét.

 

Với chuỗi có dấu tùy ý thì ta xét xem nó có hội tụ tuyệt đối không, nghĩa là xét $lim \sqrt[n]{|a_n|}$ với tiêu chuẩn căn số cauchy hoặc $lim \left | \frac{a_{n+1}}{a_n} \right |$  d'Alembert. Nếu nó hội tụ tuyệt đối thì sẽ hội tụ

Điều này bạn nói là đúng, nhưng nó không có tác dụng sửa lỗi cho bài trên, 1 chuỗi hội tụ có thể không hội tụ tuyệt đối, nhưng đã hội tụ tuyệt đối thì nó phải hội tụ, vậy theo bài trên:

 

Đặt_ $X=(x-3)^2$ _ , _ khi đó chuỗi đã cho thành $\sum_{n=1}^{+\infty} \left(-1\right)^n \dfrac{n!}{n^n} \cdot X^n$

$\lim_{n \to \infty}\dfrac{|a_{n+1}|}{|a_{n}|} = \lim_{n \to \infty}\left [ \dfrac{\left ( n+1 \right )!}{\left ( n+1 \right )^{n+1}} \cdot \dfrac{n^n}{n!} \right ] = \lim_{n \to \infty}\dfrac{1}{\left (1+\frac{1}{n}  \right )^n} = \dfrac{1}{e}$
 

Vậy khoảng hội tụ là_ ($-e$ ; $e$)___ $\Leftrightarrow -\sqrt{e} +3 < x < \sqrt{e}+3$
 
Khi_ $x=\pm \sqrt{e}+3$ _, chuỗi đã cho có dạng_ $\sum_{n=1}^{+\infty}\left ( -1 \right )^n \cdot \dfrac{n!}{n^n} \cdot e^n$
Ta có__ $\dfrac{|u_{n+1}|}{|u_{n}|} = \dfrac{e^{n+1}.\left( n+1 \right )!}{\left ( n+1 \right )^{n+1}} \cdot \dfrac{n^n}{e^n.n!} = \dfrac{e .n^n}{\left (n+1 \right )^n} = \dfrac{e}{\left (1+\frac{1}{n} \right )^n} \rightarrow 1$
 Tuy nhiên vì__$\left (1+\dfrac{1}{n}  \right )^n < e$ __nên__$\dfrac{|u_{n+1}|}{|u_{n}|} > 1$
Vậy__ $\sum_{n=1}^{+\infty}\left ( -1 \right )^n \cdot \dfrac{n!}{n^n} \cdot e^n$__phân kỳ .
 
Kết luận , miền hội tụ của chuỗi đã cho là_ ($-\sqrt{e} +3$ ; $\sqrt{e}+3$) .

Nếu bạn nói là sieumau88 đang xét hội tụ tuyệt đối nếu cứ cho là đúng thì bạn chẳng biện hộ được gì cả, đúng không? kết quả nó ra là phân kì với $x=-1$, mà phân kì không thể dùng được hội tụ tuyệt đối, bạn còn gì chỉ giáo không?? sai kiến thức cơ bản rồi đó!




#427626 Tìm miền hội tụ:$\sum\limits_{n=1}^\infty...

Đã gửi bởi letrongvan on 15-06-2013 - 20:23 trong Giải tích

Đúng là khi xét một chuỗi để biết hội tụ hay không hội tụ tuyệt đối thì xảy ra các trường hợp:

  • Nếu nó hội tụ tuyệt đối thì đơn nhiên hội tụ
  • Nếu nó không hội tụ tuyệt đối thì nó có thể bán hội tụ, nên không thể nói nó phân kì

Đồng ý với bạn về những vấn đề trên. Nhưng ở đây sieumau88 đang dùng tiêu chuẩn d'Alembert để xét tính hội tụ của $\sum_{k=0}^{+\infty} {\left | a_k \right |}$ nhờ vào $\lim_{k\rightarrow +\infty}{\left | \frac{a_{k+1}}{a_k} \right |}$, kết quả thu được là  $\sum_{k=0}^{+\infty} {\left | a_k \right |}$ phân kỳ thì có thể khẳng định được rằng $\sum_{k=0}^{+\infty} {a_k}$ cũng phân kỳ (1). Vì $\left | a_k \right |$ không tiến tới 0 khi $k\rightarrow +\infty$, nên $a_k$ cũng không tiến về 0 khi $k\rightarrow +\infty$

 

(1) Nguyễn Đình Trí - Giáo trình Toán học Cao cấp - tập 2 (trang 137)

Được rồi, phản ví dụ nhé:

$\sum (-1)^{n}.\frac{1}{n}$ là chuỗi hội tụ theo laibnit ok chưa bạn?

ta có $\sum |(-1)^{n}.\frac{1}{n}|$ phân kì vậy theo bạn thì bạn đúng hay sai khi nói $\sum_{k=0}^{+\infty} {a_k}$ cũng phân kỳ (1). Vì $\left | a_k \right |$ không tiến tới 0 khi $k\rightarrow +\infty$, nên $a_k$ cũng không tiến về 0 khi $k\rightarrow +\infty$? 

bạn trích dẫn đúng nhưng dùng không đúng! :D

---------------------------------

Bài trên bạn nói mình phán xét có lẽ là hơi nặng, vì một điều đơn giản như vậy không nên sai, nếu nửa bài trên đúng mà nửa dưới như vậy người ta đánh giá khả năng nó mới chỉ ở mức biết làm mà không biết áp dụng




#427348 Tìm miền hội tụ:$\sum\limits_{n=1}^\infty...

Đã gửi bởi letrongvan on 15-06-2013 - 06:54 trong Giải tích

Bạn hãy xem lại kiến thức của mình và đọc lại những bài ở trên, xem người ta nói gì, viết gì, đừng chỉ đọc đoạn giữa rồi phán xét.

 

Với chuỗi có dấu tùy ý thì ta xét xem nó có hội tụ tuyệt đối không, nghĩa là xét $lim \sqrt[n]{|a_n|}$ với tiêu chuẩn căn số cauchy hoặc $lim \left | \frac{a_{n+1}}{a_n} \right |$  d'Alembert. Nếu nó hội tụ tuyệt đối thì sẽ hội tụ

Bạn xem lại người ta có xét là chuỗi hội tụ tuyệt đối hay không? có chữ nào nói vậy không? nếu nói là xét hội tụ tuyệt đối mà trong khi kết quả trên là nó phân kì thì chẳng ý nghĩa gì cả, bạn mới là người xem lại kiến thức, giải tích 2 không học nhiều nhưng cái này không thể nói sai!




#427350 Tìm miền hội tụ:$\sum\limits_{n=1}^\infty...

Đã gửi bởi letrongvan on 15-06-2013 - 07:05 trong Giải tích

Thứ nhất, về mấy cái bạn bảo to tát, mình chỉ nói đó là một cách nhìn chứ không phải dùng nó để giải thích cái gì cả. Còn về $|u_{k+1}/|u_{k}|>1|$ suy ra ngay, $|u_{n}|  > |u_{n_0}|$ với $n_0$ cố định (đủ lớn để $|u_{k+1}/|u_{k}|>1|$ xảy ra). Từ đó $|u_n|$ không thể hội tụ về $0$ và do đó $u_n$ cũng không thể hội tụ về 0.

Đúng là 1 cách nhìn, không dùng giải thích,hình như bạn được học khá sâu về không gian vector,topo đại cương vì bạn trích dẫn khá nhiều khái niệm trừu tượng, trong các giáo trình thường không có




#427014 Tìm miền hội tụ:$\sum\limits_{n=1}^\infty...

Đã gửi bởi letrongvan on 14-06-2013 - 02:23 trong Giải tích

 

 

Nhìn chung, tính hội tụ của một chuỗi có thể vẫn bảo toàn nếu ta thay đổi chuỗi một chút xíu; nói trừu tượng hơn, trong một topo phù hợp của không gian các chuỗi, tập các chuỗi hội tụ là mở. Trong khi đó, tính giảm của chuỗi quá nghiêm ngặt.

 

Trong bài này, khi $|u_{k+1}|/|u_k| >1$ thì dễ thấy dãy các số hạng không hội tụ về không nên (dù đan dấu hay không) chuỗi đều phân kì.

 

Tính hội tụ của chuỗi không đổi nếu ta bớt đi 1 hữu hạn số hạng đầu hoặc thêm vào một hữu hạn số hạng. Công lực về không gian vecto vẫn còn hạn chế nên bạn nói những khái niệm topo đại cương thì quá là to tát, mà trong hầu hết các chương trình toán cao cấp họ không muốn đề cập đến mà chỉ học lý thuyết cơ bản, trừ các trường ngành toán mới chuyên sâu về lĩnh vực này, còn $|u_{k+1}|/|u_k| >1$ nó có cho thấy ý nghĩa gì không? để kết luận dãy các số hạng không hội tụ về không? Nếu bạn nói vậy hãy chứng minh, thật sự những gì bạn nói quá hàn lâm, dành cho chuyên ngành toán mới cần hiểu




#427015 Tìm miền hội tụ của chuỗi lũy thừa $\sum_{n=1}^{...

Đã gửi bởi letrongvan on 14-06-2013 - 02:30 trong Giải tích

 

 

 

 

Khi_ $x=-1$ _, chuỗi_ $\sum_{n=1}^{\infty }\dfrac{(-1)^{n+1}}{n(n+1)}$ ~ $\sum_{n=1}^{\infty }\dfrac{(-1)^{n+1}}{n^2}$ _hội tụ , _do chuỗi $\sum_{n=1}^{\infty }\dfrac{(-1)^{n+1}}{n^2}$ hội tụ .

 

Kết luận , miền hội tụ của chuỗi đã cho là_ $ [ -1 ; 1] $

Bạn xem lại kiến thức xem khi xét tiêu chuẩn so sánh này áp dụng với những chuỗi gì, về phần trên tôi không nói nhưng riêng phần này bạn sai quá nặng, lúc nào cũng áp dụng các tiêu chuẩn so sánh của chuỗi dương mà trong khi nó là chuỗi đan dấu




#427016 Tìm miền hội tụ của chuỗi lũy thừa $\sum_{n=1}^{...

Đã gửi bởi letrongvan on 14-06-2013 - 02:33 trong Giải tích

Trong khi chuỗi này chỉ cần xét hội tụ tuyệt đối là có thể ra kết quả là nó hội tụ khi x=-1




#400768 Tìm miền hội tụ của $\sum_{n=1}^{\infty }n...

Đã gửi bởi letrongvan on 28-02-2013 - 21:15 trong Giải tích

Tìm miền hội tụ các chuỗi sau:
Bài 1: $\sum_{n=1}^{\infty }n(\sqrt[n]{x})-1)^{3}$, x>0
Bài 2: $\sum_{n=1}^\infty {\frac{\sqrt[n]{x}-1}{nx}}, x>0$
Bài 3: $\sum_{n=1}^{\infty }{\frac{\sqrt[n]{x}-1}{\sqrt[3]{nx}}}, x>0$



#401545 Tìm miền hội tụ của $\sum_{n=1}^{\infty }n...

Đã gửi bởi letrongvan on 03-03-2013 - 06:53 trong Giải tích

Không ai trả lời, chán nhể :|



#423580 Tìm miền hội tụ $\sum \frac{e^{n}}{n...

Đã gửi bởi letrongvan on 03-06-2013 - 22:50 trong Giải tích

Bài 1

bán kính hội tụ $R = \lim_{x->\infty }\frac{a_{n}}{a_{n+1}}=\lim_{x->\infty}\frac{e^{n}(n+1)!}{n!e^{n+1}}=\lim_{x->\infty }\frac{n+1}{e}=\infty$

=> miền hội tụ là : $(-\infty ,\infty )$

 

Chưa làm nhưng nhìn thế này xin góp ý, chuỗi mà bạn dùng chưa là chuỗi lũy thừa nên trước khi xét giới hạn nên đặt ẩn chuyển về dạng chuỗi lũy thừa.

Những cái dưới không học nên không ý kiến thêm




#414949 Tìm m để có tổ hợp tuyến tính

Đã gửi bởi letrongvan on 26-04-2013 - 20:06 trong Đại số tuyến tính, Hình học giải tích

Dùng định nghĩa, cho x=a.u+b.v+c.w trong đó a,b,c là các số thực

 

Làm đơn giản theo đúng định nghĩa để chuyển sang giải hệ phương trình tuyến tính. Nếu em ra kết quả là không tồn tại m cũng không có gì là lạ đâu. Vì tập $\left \{ u,v,w \right \}$ không phải là cơ sở của $R^3$ nên kết quả của em chưa sai.

Cái mà anh nói không liên quan đâu anh à, nó là cơ sở hay không thì không quan trọng, chỉ cần nó là tổ hợp tuyến tính của 3 vector kia thôi mà




#414609 Tìm giới hạn bằng vô cùng bé tương đương

Đã gửi bởi letrongvan on 24-04-2013 - 17:25 trong Giải tích

Các bạn xem dùm mình bài này nhé:
$\lim_{x\rightarrow +\infty}x^{2}\left ( \frac{e^{\frac{1}{x^{2}}}-\cos \frac{1}{x}}{\arctan x} \right )$


SORRY các bạn nhé! Vô cực ở trên là dương vô cực. Mình ko biết ghi thế nào. Hehe. Bài này có dạng vô định là $\infty .0$ Mình ko biết khử dạng vô định thế nào. Vì cả Khai triển Maclaurint và VCB đề dùng khi x->0.

Nếu vậy trong trường hợp khác bạn có thể đặt t=1/x khi đó t-->0 rồi VCB có thể được dùng rồi đấy




#414608 Tìm giới hạn bằng vô cùng bé tương đương

Đã gửi bởi letrongvan on 24-04-2013 - 17:23 trong Giải tích

Cảm ơn bạn Võ Văn Đức nhiều nha!!!
Bạn cho mình hỏi nốt nhé.Trong sách giải thế này:
$\lim_{x\rightarrow +\infty}x^{2}\left ( \frac{e^{\frac{1}{x^{2}}}-\cos \frac{1}{x}}{\arctan x} \right )$ = $\lim_{x\rightarrow +\infty}x^{2}\left ( \frac{{1+\frac{1}{x^{2}}}+o\left ( \frac{1}{x^{3}} \right )-\left ( 1-\frac{1}{2x^{2}}+o\left ( \frac{1}{x^{3}} \right ) \right )}{\arctan x} \right ) =$$\lim_{x\rightarrow +\infty}x^{2}\left ( \frac{{\frac{3}{2x^{2}}+o\left ( \frac{1}{x^{3}} \right )}}{\arctan x} \right ) =$$\frac{3}{\pi }$
Vậy là giải bằng khai triển maclaurint phải không bạn?

Bài này giải sai 100% rồi, cosx khai triển maclorin được nhưng không thể thay thế khi cos(1/x) vì đây là 2 VCB không tương đương, thay vớ vẩn là sai rồi!




#451040 Tìm giới hạn $ \lim_{x \to +\infty } \frac...

Đã gửi bởi letrongvan on 16-09-2013 - 21:04 trong Giải tích

Tìm giới hạn

$ \lim_{x \to +\infty } \frac{ln(1+3^x)}{ln(1+2^x)} $

 

 Áp dụng quy tắc L'hospital $\lim_{x\rightarrow +\infty }\frac{1+3^x}{1+2^x}=\lim_{x\rightarrow +\infty }\left ( \frac{ln3}{ln2}.\frac{3^x+6^x}{2^x+6^x} \right )=\frac{ln3}{ln2}$




#451042 Tìm giới hạn $ \lim_{x \to +\infty } \frac...

Đã gửi bởi letrongvan on 16-09-2013 - 21:09 trong Giải tích

Tìm giới hạn

 

$ \lim_{x \rightarrow 3^+} \frac{1}{x+2^{\frac{1}{x-3}}}$

khi $x\rightarrow 3^{+}$ thì $2^{\frac{1}{x-3}}\rightarrow + \infty \Rightarrow \lim_{x\rightarrow 3^{+}}\frac{1}{x+2^{\frac{1}{x-3}}}=0$

-------------------

đúng không nhỉ?  :closedeyes:




#452210 Tìm giới hạn $ \lim_{x \to +\infty } \frac...

Đã gửi bởi letrongvan on 22-09-2013 - 07:47 trong Giải tích

Lắm lúc anh nhớ nhầm hoặc không nhớ đạo hàm với mấy cái tích phân cơ bản  :ukliam2:




#452213 Tìm giới hạn $ \lim_{x \to +\infty } \frac...

Đã gửi bởi letrongvan on 22-09-2013 - 07:54 trong Giải tích

 

$\lim_{x\to \infty}\frac{ln(1+3^x)}{ln(1+2^x)}=\lim_{x\to \infty}\frac{3^x.ln3.(1+x^2)}{2^x.ln2.(1+3^x)}=log_{2}3$

Có thể dùng VCL: $\lim_{x\to \infty}\frac{ln(1+3^x)}{ln(1+2^x)}=\lim_{x\to \infty}\frac{ln(3^x)}{ln(2^x)}=log_{2}3$

Em nghĩ anh nên xem lại đạo hàm! hihi

Mấy cái này hay hại não lắm! ~O)

 

 

Như này chắc không sai nữa đúng không?

 

 Áp dụng quy tắc L'hospital $\lim_{x\rightarrow +\infty }\frac{1+3^x}{1+2^x}=\lim_{x\rightarrow +\infty }\left ( \frac{ln3}{ln2}.\frac{3^x+6^x}{2^x+6^x} \right )=\frac{ln3}{ln2}$

 



#415023 Tìm cơ sở và số chiều

Đã gửi bởi letrongvan on 27-04-2013 - 08:41 trong Đại số tuyến tính, Hình học giải tích

Cũng kha khá đó, tưởng quên luôn bài này, không làm được, nhớ mãi mới ra, nhất là cái tính chất đưa về ma trận cột tìm hạng của hệ vector, tí thì quên mất




#414945 Tìm cơ sở và số chiều

Đã gửi bởi letrongvan on 26-04-2013 - 20:00 trong Đại số tuyến tính, Hình học giải tích

$u\in W, u=(x_{1},x_{2},x_{3},x_{4})$

$u=\begin{pmatrix} 1\\ 0\\ 0\\ 0 \end{pmatrix}x_{1}+\begin{pmatrix} 0\\ 1\\ 0\\ 0 \end{pmatrix}x_{2}+\begin{pmatrix} 0\\ 0\\ -1\\ 0 \end{pmatrix}x_{3}+\begin{pmatrix} 0\\ 0\\ 0\\ 2 \end{pmatrix}x_{4}$ dễ thấy 4 cái vector trong ngoặc đó là một hệ sinh của W, chỉ cần kiểm tra xem nó có độc lập tuyến tính không, nếu đã độc lập tuyến tính thì đó chính là cơ sở của W và số chiều bằng 4, nếu không thì sẽ có 1 vector là tổ hợp tuyến tính của các vector còn lại, tốt nhất bạn lập ma trận cột 4 vector đó ra và tìm 1 cơ sở hoàn chỉnh, khi đó số chiều <4.

Cái này bạn tự làm nhé :D




#437673 Tìm các giá trị riêng và véc tơ riêng của toán tử $\varphi :\m...

Đã gửi bởi letrongvan on 24-07-2013 - 03:31 trong Đại số tuyến tính, Hình học giải tích

Câu 1: viết ma trận của toán tử tuyến tính ra nó là cái này:

$A=\begin{pmatrix} 4 & 5 & 6 & 1\\ -5 & -7 & -9 & 1\\ 2& 3 & 4 &1 \\ -1& 2& -5&1 \end{pmatrix}$ rồi viết đa thức đặng trưng cho nó bằng không$detA=det\begin{pmatrix} 4-\lambda & 5 & 6 & 1\\ -5 & -7- \lambda& -9 & 1\\ 2& 3 & 4 -\lambda&1 \\ -1& 2& -5&1- \lambda \end{pmatrix}=0$ $\lambda$ là giá trị riêng rồi tìm vector riêng.

Cầu 2: không học nên không làm, đọc lại kiến thức trong giáo trình xem, chắc không khó lắm đâu

câu 3: 

à, chuỗi số dương, dùng tiêu chuẩn cố-sĩ với chú lý $lim_{n\rightarrow +\infty }\sqrt[n]{a}=1$ ở đây cho $a>0$ cho nó chặt, khi đó $lim_{n\rightarrow +\infty }\sqrt[n]{u_{n}}=0$ suy ra chuỗi a hội tụ.

b. bạn xét $|u_{n}|$ khi đó nó là chuỗi dương, $3^{2n+1}=3.9^{n}$ m $\frac{7^{n}}{3.9^{n}}=\frac{1}{3}.(\frac{7}{9})^{n}$ ta có $ lim_{n\rightarrow \infty }\sqrt[n]{\frac{1}{3}.(\frac{7}{9})^{n}}=\frac{7}{9}$ vậy chuỗi đã cho hội tụ theo cô-si.

bạn kiểm tra lại giúp